Solving Particular Integral of Polynomial & Exponential

  • Thread starter Thread starter enosthapa
  • Start date Start date
  • Tags Tags
    Integral
Click For Summary
The discussion focuses on solving the particular integral (PI) of the differential equation y'' + 5y' + 4y = x^2 + 2e^(-x). Participants highlight that when the right-hand side (RHS) includes an exponential term that is also a solution of the left-hand side (LHS), it cannot be used as a PI by itself. Instead, it is suggested to multiply the exponential term by x, resulting in a trial solution of the form xe^(-x) combined with a polynomial. There is some confusion regarding how to incorporate both the polynomial and exponential components into the PI. The conversation emphasizes the need to adjust the trial solution to account for the presence of both types of terms on the RHS.
enosthapa
Messages
9
Reaction score
0
y''+5y'+4y=x^2+2e^(-x)
Auxiliary roots: m1=-1 m2=-4

Using trial please
cant seem to work out PI with polynomial and exponential together.
 
Physics news on Phys.org
hi enosthapa! :smile:

(try using the X2 icon just above the Reply box :wink:)

when your RHS contains an exponential which is a solution of the LHS, it obviously can't be a PI on its own (as you've found out! :redface:) …

you need to multiply it by x (or xn is it's an nth root of the LHS) …

in this case, try xe-x (plus a polynomial) :wink:
 
Thanks a lot but I am still confused
As far as i know,
if there is exponential on RHS (ekx) and if 'k' is simple root of the auxiliary equation which it is in this case, then we try a.x.ek.x i.e a.x.e-x

but the question has polynomial in addition so did you mean i have to add the 'PI' I would get if there was polynomial only on RHS?
sorry i m confusing u:/
 
i meant a polynomial plus a constant times xe-x

doesn't that work? :confused:
 
I don't have the answer which could have been helpful. But thanks anyway.
 
Moderator's note: thread moved from "Differential Equations".

The usual rules rules for giving homework help are now in effect.
 
Question: A clock's minute hand has length 4 and its hour hand has length 3. What is the distance between the tips at the moment when it is increasing most rapidly?(Putnam Exam Question) Answer: Making assumption that both the hands moves at constant angular velocities, the answer is ## \sqrt{7} .## But don't you think this assumption is somewhat doubtful and wrong?

Similar threads

Replies
7
Views
2K
Replies
3
Views
2K
Replies
4
Views
2K
  • · Replies 10 ·
Replies
10
Views
2K
  • · Replies 5 ·
Replies
5
Views
2K
  • · Replies 19 ·
Replies
19
Views
2K
  • · Replies 6 ·
Replies
6
Views
1K
  • · Replies 4 ·
Replies
4
Views
2K
Replies
20
Views
2K
Replies
9
Views
2K